Почему спиральность безмассовых частиц лоренц-инвариантна?

По определению спиральность — это проекция спина на 3-импульс.

час "=" Дж п
где п "=" ( п 1 , п 2 , п 3 ) - оператор импульса и Дж "=" ( Дж 1 , Дж 2 , Дж 3 ) угловой оператор.

Теперь при преобразовании Лоренца безмассовые частицы преобразуются следующим образом:

U ( Λ ) | п , о "=" е я θ о | Λ п , о .

Как мы видим, импульс меняется, а спин нет.

Предположим, что состояние | п , о это состояние спиральности о такой, что у нас есть

час | п , о "=" Дж 3 п 3 | п , о "=" о п 3 | п , о

Но для государства U ( Λ ) | п , о "=" е я θ о | Λ п , о , мы бы хотели иметь

час | Λ п , о "=" о п 3 е я θ о | Λ п , о |
Таким образом, для сохранения спиральности нам потребуется п 3 "=" п 3 что не всегда так.

Так почему же говорят, что спиральность инвариантна по Лоренцу?

Ответ в приведенной вами ссылке хорош, но определение спиральности другое.

Ответы (1)

Ваша формула для оператора спиральности неверна ; это должно быть ясно уже на уровне размерного анализа. Правильная формула (см. ссылки 1 и 2)

час "=" Дж п | п |
где | п | обозначает норму п . Воздействуя на свое состояние с помощью час не дает факторов п 3 , так что "парадокс" разрешился.

Использованная литература.

  1. Шварц - Квантовая теория поля и Стандартная модель §11.1.

  2. Тиччиати - Квантовая теория поля для математиков §7.8.

@amiltonmoreira Можешь попробовать еще раз, пожалуйста? Я понятия не имею, что ты пытаешься сказать. (Сначала у вас есть оператор, затем состояние, затем тензорное произведение состояний; ни одно из них не может быть равно друг другу...)
извините проблема с компьютером
Предположим, что у нас есть Дж п | п | | п , о "=" о | п , о . Теперь предположим, что наше преобразование представляет собой поворот в Икс ось, чем Дж п | п | | Λ п , о "=" ( п 2 Дж 2 п 3 + п 3 о п 3 ) | Λ п , о . Является ( п 2 Дж 2 п 3 + п 3 о п 3 ) | Λ п , о равно о | Λ п , о ?
Дж п | п | очевидно, является скаляром при вращении ... Эффективность не может меняться при вращении, это также верно, если частица массивна.
Как я могу это доказать?
Просто примените унитарное представление вращений и используйте тот факт, что Дж и п являются 3-векторами под его действием, тогда как | п | это скаляр....